Saturday, September 24, 2011

IMO 2011 Problems and Solutions

International Mathematical Olympiad 2011


Problems
Problem 1: Given any set A = \{a_1, a_2, a_3, a_4\} of four distinct positive integers, we denote the sum a_1 +a_2 +a_3 +a_4 by s_A. Let n_A denote the number of pairs (i, j) with 1 \leq  i < j \leq 4 for which a_i +a_j divides s_A. Find all sets A of four distinct positive integers which achieve the largest possible value of n_A.

Proposed by Fernando Campos, from Mexico
Problem 2: Let \mathcal{S} be a finite set of at least two points in the plane. Assume that no three points of \mathcal S are collinear. A windmill is a process that starts with a line \ell going through a single point P \in \mathcal S. The line rotates clockwise about the pivot P until the first time that the line meets some other point belonging to \mathcal S. This point, Q, takes over as the new pivot, and the line now rotates clockwise about Q, until it next meets a point of \mathcal S. This process continues indefinitely.
Show that we can choose a point P in \mathcal S and a line \ell going through P such that the resulting windmill uses each point of \mathcal S as a pivot infinitely many times.

Proposed by Geoffrey Smith, United Kingdom
 
Problem 3: Let f : \mathbb R \to \mathbb R be a real-valued function defined on the set of real numbers that satisfies
f(x + y) \leq yf(x) + f(f(x))
for all real numbers x and y. Prove that f(x) = 0 for all x \leq 0.

Proposed by Igor Voronovich, Belarus
 
Problem 4: Let n > 0 be an integer. We are given a balance and n weights of weight 2^0, 2^1, \cdots, 2^{n-1}. We are to place each of the n weights on the balance, one after another, in such a way that the right pan is never heavier than the left pan. At each step we choose one of the weights that has not yet been placed on the balance, and place it on either the left pan or the right pan, until all of the weights have been placed.
Determine the number of ways in which this can be done.

Proposed by Morteza Saghafian, Iran
 
Problem 5: Let f be a function from the set of integers to the set of positive integers. Suppose that, for any two integers m and n, the difference f(m) - f(n) is divisible by f(m- n). Prove that, for all integers m and n with f(m) \leq f(n), the number f(n) is divisible by f(m).

Proposed by Mahyar Sefidgaran, Iran  
 
Problem 6: Let ABC be an acute triangle with circumcircle \Gamma. Let \ell be a tangent line to \Gamma, and let \ell_a, \ell_b and \ell_c be the lines obtained by reflecting \ell in the lines BC, CA and AB, respectively. Show that the circumcircle of the triangle determined by the lines \ell_a, \ell_b and \ell_c is tangent to the circle \Gamma.

Proposed by Japan

No comments:

Post a Comment